Is x < 0?

This topic has expert replies
User avatar
Elite Legendary Member
Posts: 3991
Joined: Fri Jul 24, 2015 2:28 am
Location: Las Vegas, USA
Thanked: 19 times
Followed by:37 members

Is x < 0?

by Max@Math Revolution » Thu Sep 13, 2018 11:33 pm

Timer

00:00

Your Answer

A

B

C

D

E

Global Stats

[Math Revolution GMAT math practice question]

Is x < 0?

1) |x|=-x
2) |x|>x

GMAT/MBA Expert

User avatar
GMAT Instructor
Posts: 16207
Joined: Mon Dec 08, 2008 6:26 pm
Location: Vancouver, BC
Thanked: 5254 times
Followed by:1268 members
GMAT Score:770

by Brent@GMATPrepNow » Fri Sep 14, 2018 5:46 am

Timer

00:00

Your Answer

A

B

C

D

E

Global Stats

Max@Math Revolution wrote:
Is x < 0?

1) |x| = -x
2) |x| > x
Target question: Is x < 0?

Statement 1: |x| = -x
Let's TEST some values.
There are several values of x that satisfy statement 1. Here are two:
Case a: x = -1. Notice that |-1| = -(-1). In this case, the answer to the target question is YES, x IS less than 0
Case b: x = 0. Notice that |0| = -0. In this case, the answer to the target question is NO, x is NOT less than 0
Since we cannot answer the target question with certainty, statement 1 is NOT SUFFICIENT

Statement 2: |x| > x
If |x| > x, then we can be certain that x does not equal 0.
So, let's see what happens if x is POSITIVE, and what happens if x is NEGATIVE

If x is positive, then |x| > x becomes |POSITIVE| > POSITIVE
This doesn't work, because |some POSITIVE number| is always equal to that same number.
For example, |3| = 3 and |12.9| = 12.9
So, if x is POSITIVE, it cannot be the case that |x| > x
In other words, x CANNOT by positive

So, if x does not equal 0 and x CANNOT by positive, then we can be certain that x is negative
In other words, the answer to the target question is YES, x IS less than 0
Since we can answer the target question with certainty, statement 2 is SUFFICIENT

Answer: B

Cheers,
Brent
Brent Hanneson - Creator of GMATPrepNow.com
Image

User avatar
GMAT Instructor
Posts: 1449
Joined: Sat Oct 09, 2010 2:16 pm
Thanked: 59 times
Followed by:33 members

by fskilnik@GMATH » Fri Sep 14, 2018 6:39 am

Timer

00:00

Your Answer

A

B

C

D

E

Global Stats

Max@Math Revolution wrote:[Math Revolution GMAT math practice question]

Is x < 0?

1) |x|=-x
2) |x|>x
\[x\,\,\mathop < \limits^? \,\,0\]

\[\left( 1 \right)\,\,\,\left| x \right| = - x\,\,\,\,\, \Leftrightarrow \,\,\,\,x \leqslant 0\,\,\,\,\left\{ \begin{gathered}
\,{\text{Take}}\,\,x = 0\,\,\,\, \Rightarrow \,\,\,\,\left\langle {{\text{NO}}} \right\rangle \hfill \\
\,{\text{Take}}\,\,x = - 1\,\,\,\, \Rightarrow \,\,\,\,\left\langle {{\text{YES}}} \right\rangle \hfill \\
\end{gathered} \right.\]
\[\left( 2 \right)\,\,\,x < \left| x \right|\,\,\,\,\,\,\mathop \Leftrightarrow \limits^{\left( * \right)} \,\,\,\,\,x < 0\,\,\,\,\,\,\, \Rightarrow \,\,\,\,\left\langle {{\text{YES}}} \right\rangle \,\,\,\,\,\,\,\,\, \Rightarrow \,\,\,\,{\text{SUFF}}{\text{.}}\,\,\,\,\]
\[\left( * \right)\,\,x \geqslant 0\,\,\,\, \Rightarrow \,\,\,x = \left| x \right|\]


This solution follows the notations and rationale taught in the GMATH method.

Regards,
Fabio.
Fabio Skilnik :: GMATH method creator ( Math for the GMAT)
English-speakers :: https://www.gmath.net
Portuguese-speakers :: https://www.gmath.com.br

User avatar
Elite Legendary Member
Posts: 3991
Joined: Fri Jul 24, 2015 2:28 am
Location: Las Vegas, USA
Thanked: 19 times
Followed by:37 members

by Max@Math Revolution » Sun Sep 16, 2018 5:25 pm

Timer

00:00

Your Answer

A

B

C

D

E

Global Stats

=>

Forget conventional ways of solving math questions. For DS problems, the VA (Variable Approach) method is the quickest and easiest way to find the answer without actually solving the problem. Remember that equal numbers of variables and independent equations ensure a solution.

Since we have 1 variable (x) and 0 equations, D is most likely to be the answer. So, we should consider each of the conditions on their own first.

Condition 1)
|x| = -x
=> x ≤ 0
In inequality questions, the law "Question is King" tells us that if the solution set of the question includes the solution set of the condition, then the condition is sufficient.
Since the solution set "x<0" of the question doesn't include the solution set "x ≤ 0" of the condition 1), condition 1) is not sufficient.

Condition 2)
|x|>x
=> x < 0
Since condition 2) is equivalent to the question, condition 2) is sufficient.

Therefore, B is the answer.

Answer: B

If the original condition includes "1 variable", or "2 variables and 1 equation", or "3 variables and 2 equations" etc., one more equation is required to answer the question. If each of conditions 1) and 2) provide an additional equation, there is a 59% chance that D is the answer, a 38% chance that A or B is the answer, and a 3% chance that the answer is C or E. Thus, answer D (conditions 1) and 2), when applied separately, are sufficient to answer the question) is most likely, but there may be cases where the answer is A,B,C or E.